What is Angular momentum: Definition and 1000 Discussions

In physics, angular momentum (rarely, moment of momentum or rotational momentum) is the rotational equivalent of linear momentum. It is an important quantity in physics because it is a conserved quantity—the total angular momentum of a closed system remains constant.
In three dimensions, the angular momentum for a point particle is a pseudovector r × p, the cross product of the particle's position vector r (relative to some origin) and its momentum vector; the latter is p = mv in Newtonian mechanics. Unlike momentum, angular momentum depends on where the origin is chosen, since the particle's position is measured from it.
Just as for angular velocity, there are two special types of angular momentum of an object: the spin angular momentum is the angular momentum about the object's centre of mass, while the orbital angular momentum is the angular momentum about a chosen center of rotation. The total angular momentum is the sum of the spin and orbital angular momenta. The orbital angular momentum vector of a point particle is always parallel and directly proportional to its orbital angular velocity vector ω, where the constant of proportionality depends on both the mass of the particle and its distance from origin. The spin angular momentum vector of a rigid body is proportional but not always parallel to the spin angular velocity vector Ω, making the constant of proportionality a second-rank tensor rather than a scalar.
Angular momentum is an extensive quantity; i.e. the total angular momentum of any composite system is the sum of the angular momenta of its constituent parts. For a continuous rigid body or a fluid the total angular momentum is the volume integral of angular momentum density (i.e. angular momentum per unit volume in the limit as volume shrinks to zero) over the entire body.
Torque can be defined as the rate of change of angular momentum, analogous to force. The net external torque on any system is always equal to the total torque on the system; in other words, the sum of all internal torques of any system is always 0 (this is the rotational analogue of Newton's Third Law). Therefore, for a closed system (where there is no net external torque), the total torque on the system must be 0, which means that the total angular momentum of the system is constant. The conservation of angular momentum helps explain many observed phenomena, for example the increase in rotational speed of a spinning figure skater as the skater's arms are contracted, the high rotational rates of neutron stars, the Coriolis effect, and the precession of gyroscopes. In general, conservation limits the possible motion of a system but does not uniquely determine it.
In quantum mechanics, angular momentum (like other quantities) is expressed as an operator, and its one-dimensional projections have quantized eigenvalues. Angular momentum is subject to the Heisenberg uncertainty principle, implying that at any time, only one projection (also called "component") can be measured with definite precision; the other two then remain uncertain. Because of this, the axis of rotation of a quantum particle is undefined. Quantum particles do possess a type of non-orbital angular momentum called "spin", but this angular momentum does not correspond to a spinning motion.

View More On Wikipedia.org
  1. jaumzaum

    Conservation of angular momentum and tangential velocity

    I have 2 questions about the situations below http://img594.imageshack.us/img594/1/sadgsfdg.png Let vcp be the initial tangential velocity required for the upper body for that, when the low body is released, the upper body describes a circular trajectory I - In situation A, if we leave...
  2. S

    Hamiltonian for 2 Particles with Angular Momentum

    Homework Statement The Hamiltonian for two particles with angular momentum j_1 and j_2 is given by: \hat{H} = \epsilon [ \hat{\bf{j}}_1 \times \hat{\bf{j}}_2 ]^2, where \epsilon is a constant. Show that the Hamiltonian is a Hermitian scalar and find the energy spectrum.Homework Equations...
  3. S

    Total Angular Momentum Commutation Relations for 2 Particle

    Hey, I'm not exactly sure how much this question wants, however the two in question are parts a) and b) below. So part a) asks to write the expression for the total angular momentum J, I though this was just: \hat{J}=\hat{J}^{(1)}+\hat{J}^{(2)} but when we come to showing it...
  4. A

    Expectation values for angular momentum

    Consider a quantum system with angular momentum 1, in a state represented by the vector \Psi=\frac{1}{\sqrt{26}}[1, 4, -3] Find the expectation values <L_{z}> and <L_{x}> I'm reviewing my quantum mechanics; I had a pretty horrible course on it during undergrad. I feel like this should be...
  5. M

    Angular momentum vectors and frisbees

    While working on shooting mechanisms for the 2013 FIRST Robotics Competition, our mentoring scientist noted that the direction of the angular momentum vector pointed down for our prototype shooter. So now we're debating whether the vector points up or down matters for the flight of the frisbee...
  6. S

    Uncertainties & Total Angular Momentum

    Hey, My question is on determining an 'uncertainty' quantity using total angular momentum operators in the x,y and z directions where we know the commutation relations between the x,y and z directions of the total angular momentum operators. I'm not really sure where to go with this at...
  7. P

    Angular momentum; turntable problem

    Homework Statement A 1.7 kg , 20cm--diameter turntable rotates at 140rpm on frictionless bearings. Two 480g blocks fall from above, hit the turntable simultaneously at opposite ends of a diameter, and stick. What is the turntable's angular velocity, in rpm , just after this event...
  8. S

    Possible values of total angular momentum of a 2 electron system

    Hey, Also a bit confused on this one, the question is displayed below The possible total spin values are 1,0,-1 and I know that \mid l-s\mid\leq j\leq l+s\: ,\: -j\leq m_{z}\leq j Where the latter inequality is in integer steps, so I'm not really sure if it is as simple as...
  9. S

    Eigenvalue of Total Angular Momentum Probability

    Hey, My question is on the probability of attaining a particular eigenvalue for the total angular momentum operator squared for a particular state ψ, the question is shown in the image below: I believe the eigenvalue of the total angular momentum operator squared is given by j(j+1)...
  10. G

    Angular momentum and energy conservation.

    Homework Statement A mass m is laying on a frictionless table and is connected to a mass M with a nonelastic string going through the center of the table. At t=0 the m mass is at a r0 distance from the center of the table, and is moving at a v0 velocity in the tangent direction. Find the...
  11. D

    Confused about direction in angular momentum

    Homework Statement Alright apologies I could not attach an image due to my inexperience of this but its simple to imagine. Imagine a stick which can rotate about the centre of its axis, perpendicular to its length. I.e the ends can rotate freely in a clockwise, or anti clockwise manner...
  12. ssamsymn

    Classical Angular Momentum

    [Li,Lj]=εijkLk how can I prove this expression classically?
  13. R

    [QM] Total angular momentum rotation operator

    Homework Statement How to prove that for any representation of the spin, the state e^{-i{\pi}J_x/\hbar}|j,m\rangle is proportional to |j,-m\rangle The exponential term is the rotation operator where J_x is the x-component of the total angular momentum operator, and |j,m\rangle is an...
  14. barryj

    Linear and angular momentum problem

    This is not a homework problem. I am trying to set up the following problem. I am doing something wrong. Help. I have attached the problem and figure but here is the text. Figure 10-52 shows a thin, uniform bar of Length L and mass M and a small blob of putty of mass m. The system is...
  15. D

    Spin angular momentum of electron

    hi pf, i got following questions in my mind while reading about the spin angular momentum of an electron. 1. From rotational dynamics we know that to have angular momentum a body must necessarily rotation or motion? so does an electron rotates as it has spin angular momentum or does it...
  16. B

    Poisson brackets of angular momentum components

    I want to find [M_i, M_j] Poisson brackets. $$[M_i, M_j]=\sum_l (\frac{\partial M_i}{\partial q_l}\frac{\partial M_j}{\partial p_l}-\frac{\partial M_i}{\partial p_l}\frac{\partial M_j}{\partial q_l})$$ I know that: $$M_i=\epsilon _{ijk} q_j p_k$$ $$M_j=\epsilon _{jnm} q_n p_m$$ and so...
  17. S

    Field Angular Momentum (Thomson Dipole)

    Homework Statement I have attached the question as jpg to this post. Typing these were too time consuming and I uploaded the relevant sections as image files, thanks for your understanding. Homework Equations mu_0*epsilon_0*Poynting Vector = Momentum density Position vector X...
  18. K

    Calculating Angular Momentum of a Rotating Rod | Mass M, Angular Velocity ш

    What is the angular momentum of a rod rotating about one end (mass M and angular velocity ш),about its center of mass?
  19. S

    Angular momentum conservation in helicopters

    I had read in a book that the primary reason for the use of tail rotor in a helicopter is to counteract the rotation of the main body generated as a response to the rotation of the main rotor blades to keep angular momentum of the rotor-body system zero. Is it true that the rotation of main...
  20. T

    Angular Momentum of an object with respect to a moving point

    Homework Statement Two particles A and B having equal masses m are rotating around a fixed point O with constant angular speed ω .A is connected to point O with a string of length L/2 whereas B is connected to point A with string of length L/2 .Find the angular momentum of B with respect to...
  21. M

    Conservation of angular momentum

    Hi, I am by now very used to the concept of conservation of momentum and energy, however i had never solved a problem using the conservation of angular momentum. Well, in the case where angular momentum is conserved, we do not have a torque net. But, how this conservation law helps us to...
  22. M

    Angular momentum and coordinates

    let us denote the joint eigenstate of \hat{L^{2}}and \hat{L_{z}} by ll,m> and we know that if we are in spherical coordinates, \hat{L^{2}} and \hat{L_{z}} depend on θ and ∅, so we denote the joint eigenstate by: <θ∅l l,m>.. why?
  23. C

    Angular momentum - Planet exercise

    Homework Statement Dear all, This is my first post and I need some help. The exercise I am trying to solve is this one: A star has a radius of 6 × 10^8 m and a period of rotation of 30 days. Eventually it becomes a neutron star with a radius of 10^4 m and a period of 0.1 s. If the...
  24. P

    Kinetic energy involving angular momentum and inertia

    Homework Statement Problem with diagram here Homework Equations I=(1/3)MR2 (I believe the formula given in the problem is incorrect since it is pivoting on one end rather than the center of mass) KE=(1/2)Iω2+(1/2)Mv2 L=Iω=MvR The Attempt at a Solution a. True, the velocity is still going in...
  25. N

    Frictional Forces on a Rotating System and Disk: Understanding Angular Momentum

    Homework Statement Suppose the system is rotating clockwise when the metal disk is dropped on it. The direction of the frictional force is... zero on the disk and the system counter-clockwise on the disk, clockwise on the system clockwise on the disk, zero on the system...
  26. A

    How does Angular Momentum relate to the rapid rotation of Pulsars?

    Hello, Does the Conservation of Angular Momentum have any relation to the rapid rotation of Pulsars? I've reasoned that the Angular Momentum of the "parent" star would be concentrated on the smaller pulsar and would produce a greater frequency of rotation. Thanks,
  27. A

    Conservation of Angular Momentum and the Role of Coordinate Systems

    Imagine i have a particle in the xy plane rotating in a circle around the z-axis at a constant angular velocity. The angular momentum is r cross p which would give me a constant value at all times, there is no change and it's conserved. However if I was to move the rotating particle higher up...
  28. A

    Conversion of translational momentum to angular momentum

    Suppose there was a mass, m1, traveling rightwards at v0. There is a second mass, m2, above the line of motion of m1, and which has a rigid, massless bar of length L attached to its center of mass. This bar extends downwards so that the terminal intersects with the path of m1. When m1 reaches...
  29. B

    Understanding Zero Angular Momentum

    Hello, I am currently reading about angular momentum. A sentence in my textbook sort of confused me: "when the translational velocity of the particle is along a line that passes through the axis, the particle has zero angular momentum with respect to the axis." I was wondering if someone...
  30. G

    Angular Momentum for Man and Bullet

    Homework Statement A man, mass M, stands on a massless rod which is free to rotate about its center in the horizontal plane. The man has a gun (massless) with one bullet, mass m. He shoots the bullet with velocity Vb, horizontally. Find the angular velocity of the man as a function of the angle...
  31. E

    Orbital angular momentum possible values of an electron in a hydrogen atom

    Homework Statement What are the orbital angular momentum possible values for an electron in a hydrogen atom with a principal quantum number n = 3? Homework Equations L = \sqrt{l(l+1)}\hbar The Attempt at a Solution Possible values for l are 0, 1, 2. So, substituting these in...
  32. L

    Quick question on the conservation of angular momentum

    okay, i realize that the angular momentum of a moving point mass could be looked at about any point, and that angular momentum is conserved as long as no torque is acted on that point mass. but, something i don't understand is how, then, the angular velocity could increase if the moment of...
  33. R

    Sum of Torque vs. Conservation of Angular Momentum Quick help needed

    Number 11) This is what I did: Ʃτ = F2r2 + F1r1 = 0 (195)(7) + F1(0.7) = 0 F1(0.7) = -(195)(7) F1 = -1365/0.7 F1 = -1950 N F1 = 1950 N Is that answer right? Number 12) This is what I did: Since this is a massless rod and the location of the axis is through the end, I = ML2 Linitial =...
  34. T

    How Is Angular Momentum Calculated in Particle Motion?

    Homework Statement A 1.47kg particle moves in the xy plane with a velocity of v = (4.59i - 3.28j)m/s. Determine the magnitude of the particle's angular momentum when its position vector is r = (1.35i + 2.57j)m. Homework Equations p = mv L = r x p (the x is supposed to be a cross...
  35. S

    Obtaining the angular momentum state of two spin 1/2 particles for (S=0)

    Homework Statement consider the possible angular momentum states |s,m>, of a system of two spin-1/2 particles construct all possible states with total spin zero (S=0) Homework Equations The Attempt at a Solution if total S of system is zero, m must also equal zero. So the...
  36. A

    Black hole accretion: angular momentum loss

    Why exactly is it necessary for angular momentum to be lost by a mass if it is to accrete around a black hole? the mass is decreasing its radius, so it speeds up: thus angular mometum is conserved. But everywhere it is saying that 99.99% of the angular momentum must be shed for accretion to...
  37. T

    Calculating Moment of Inertia from Ball Angular Momentum Problem?

    Homework Statement A 0.478-kg ball is thrown at a 22.3-cm tall, 0.383-kg object and hits with a perfectly horizontal velocity of 12.8 m/s. Suppose the ball strikes at the very top of the object. After collision the ball has a horizontal velocity of 4.6 m/s in the same direction. The object...
  38. C

    Deriving relation between angular momentum reduction by torque

    Friction causes a torque that opposes angular momentum. It gets reduced. how can we derive a relation connecting these. friction starts with maximum and becomes zero. angular velocity and its corresponding momentum decreases maximum in the beginning and increases towards the end. the radial...
  39. K

    Angular momentum problem giving me a headache

    Homework Statement A heavy stick of length L = 3.3 m and mass M = 20 kg hangs from a low-friction axle. A bullet of mass m = 0.014 kg traveling at v = 117 m/s strikes near the bottom of the stick and quickly buries itself in the stick. Just after the impact, what is the angular speed ω of...
  40. S

    Point of application of angular momentum

    I know the approach is to use conservation of angular momentum. I also know that I am entirely free to pick which point I would like to calculate my L about. (Although calculating L about the CM will make it much easier, but I'd like to try otherwise) So, if I were to calculate my L(after...
  41. R

    Calculate angular momentum? I almost got the right answer but it's wrong Help

    Calculate angular momentum? I almost got the right answer but it's wrong! Help! This is what I did for part a... 1. What is given Particle mp = 0.450 kg ω = 2.00 rad/s r = 1 m Stick ms = 0.125 kg ω = 2.00 rad/s r = 0.5 m 2. Angular momentum is conserved (Linitial = Lfinal)...
  42. R

    Determining the work done on the puck using conservation of angular momentum? Help

    Determining the work done on the puck using conservation of angular momentum?? Help! This is what I did... 1) Given mpuck = 0.300 kg rinitial = 0.4 m vinitial = 0.6 m/s mpuck = 0.300 kg rfinal = 0.15 m vfinal = ____ m/s ƩW = KEfinal - KEinitial 2) KEinitial = (1/2)mv2...
  43. K

    Angular momentum: rotating rod and a ball

    Homework Statement A rod of length l swivels around an axis, denoted O in the drawing, located on the floor. The initial position is β=45°. A ball B is left to fall at the same time the rod is left to rotate. Who will arrive, first, to the floor: the ball or the edge A of the rod...
  44. S

    Successive Measurements of angular momentum

    Hello, I believe this to be a rather simple problem but I am not quite sure if my thinking is correct. We have a particle in a j=1 state of angular momentum J. I am first asked to find some eigenvectors of the matrix J(y): J_{y}=\frac{\hbar}{\sqrt{2}i}\begin{pmatrix} 0 & -1 & 0\\ 1 & 0...
  45. P

    Solving Angular Momentum Problem: Dividing by L/4 Explained

    I got a homework problem the other day, and it was a conservation of angular momentum problem. Basically a bullet hits a rod, and a rod starts to spin. I needed to find how fast the rod was rotating. I didn't get the answer right, but I was looking up the answers, and it says that to convert...
  46. C

    Spin and Orbital Angular Momentum

    Homework Statement We have a system of 2 indistinguishable spin-1 bosons. We shall adopt the center of mass frame. Let S = total spin L = relative orbital angular momentum J = L+S = total angular momentum Prove that J = 2m where m is an integer. If given that J=1, what are the permissible...
  47. N

    Energy, Angular Momentum, Torque, solid ball rolling down loop track? help?

    Energy, Angular Momentum, Torque, solid ball rolling down loop track? help!? A solid brass ball of mass .280g will roll smoothly along a loop-the-loop track when released from rest along the straight section. The circular loop has radius R = 14.0 cm, and the ball has radius r<<R. (a) What is...
  48. N

    Hollow sphere, angular momentum, torque problem? help?

    Hollow sphere, angular momentum, torque problem? help?? a hollow sphere or radius 0.15m with rotational inertia = 0.040 kg m^2 about a line through its center of mass, rolls without slipping up a surface inclined 30 degree to the horizontal. at a certain initial position,the sphere's total...
  49. C

    Quantum Mechanics: Question on Angular Momentum

    Homework Statement Consider a system that is initially in the state: \psi\left(\theta,\phi\right)=\frac{1}{\sqrt{5}}Y_{1,-1}\left(\theta,\phi\right) + \frac{\sqrt{3}}{5}Y_{1,0}\left(\theta,\phi\right)+\frac{1}{\sqrt{5}}Y_{1,1}\left(\theta,\phi\right) Part 1: Find <\psi|L_{+}|\psi> Part 2...
  50. D

    Two astronauts angular momentum unkown r

    Two astronauts, each having a mass M are connected by a length of rope of length d have a negligible mass. They are isolated in space, orbiting their center of mass at an angular speed of ω0. By pulling on the rope, one of the astronauts shortens the total distance between them to 0.668d...
Back
Top